Сұрақ #1472 Берілген бес жауаптың біреуі дұрыс

Қозғалыстар х1=3t-t2; x2=2t+t2 теңдеумен өрнектелсе, олардың бастапқы жылдамдықтары

Шешім
0%
0%
0%
0%
0%
Iздеу